I don't see how the correct answer is necessary
The conclusion is just that Canadian aboriginals used canoes made of the wood. If you negate the ...
AndrewArabie on November 15, 2022
  • December 2017 LSAT
  • SEC1
  • Q21
1
Reply
A vs D
How would D, the mortgage being an unusally large loan, be incorrect (or less right than A)?
j.k.e12 on April 19, 2022
  • December 2017 LSAT
  • SEC1
  • Q19
1
Reply
Why is E incorrect?
E states: "Some songwriters earn money solely from the radio airplay of their songs." The passage...
christina.agramonte on January 13, 2022
  • December 2017 LSAT
  • SEC1
  • Q13
1
Reply
C?
Hello, Can someone explain why the strongest support is C? I'm not understanding this one at ...
lerondagates on July 30, 2021
  • December 2017 LSAT
  • SEC1
  • Q23
6
Replies
Why E is incorrect?
The argument of C and E seems very similar to me, both looks right. Please explain:)
Margaret22 on January 21, 2021
  • December 2017 LSAT
  • SEC1
  • Q25
2
Replies
B v. C
Why is it B and not C? If the debate coach's argument depends on the assumption that Roberts ...
Sidra on January 20, 2021
  • December 2017 LSAT
  • SEC1
  • Q14
2
Replies
Why is A correct.
No where does it say that employees do not perform equally well as Ted. It does. Mention they mus...
Irene-Vera on November 9, 2020
  • December 2017 LSAT
  • SEC1
  • Q9
4
Replies
Why is the correct answer B?
Having trouble getting to the correct answer.
cjahangiri on October 9, 2020
  • December 2017 LSAT
  • SEC1
  • Q24
3
Replies
Answer C
can't having an effect on health also mean a correlation? does it necessarily imply causation? I ...
schicago on August 21, 2020
  • December 2017 LSAT
  • SEC1
  • Q4
1
Reply
Why isn't D right?
I narrowed it to D and E and then chose D.
ruchitaj on July 29, 2020
  • December 2017 LSAT
  • SEC1
  • Q8
1
Reply
explanation that could help me understand this ...
Explanation
ulino23 on July 4, 2020
  • December 2017 LSAT
  • SEC1
  • Q11
2
Replies
Answer explanation
I get why b is right, but why is a not right? In order to measure the lightening, wouldn’t the hi...
wills on June 24, 2020
  • December 2017 LSAT
  • SEC1
  • Q17
1
Reply
Why is E incorrect?
E seems to be true?
Skyler-Simon on June 23, 2020
  • December 2017 LSAT
  • SEC1
  • Q16
1
Reply
Please explain
Why is D the correct answer?
Sidra on June 13, 2020
  • December 2017 LSAT
  • SEC1
  • Q20
2
Replies
Difference between B & E?
Can you explain what the difference between B & E is? I was debating between these two and chose ...
josettebrooksbank on June 13, 2020
  • December 2017 LSAT
  • SEC1
  • Q18
3
Replies
Please explain
Why is C the correct answer?
Sidra on June 13, 2020
  • December 2017 LSAT
  • SEC1
  • Q10
2
Replies
Why is D Incorrect
Songwriters get much of the money they earn from their songs from radio airplay. {A hit song is p...
Sam1292 on April 17, 2020
  • December 2017 LSAT
  • SEC1
  • Q13
1
Reply
Confused
Can you explain why A is the correct answer and why D is incorrect? My thought process is tha...
Sam1292 on April 17, 2020
  • December 2017 LSAT
  • SEC1
  • Q7
1
Reply
I chose D...
It seemed like it could be the right answer based mostly on my process of elimination, however I ...
Christy-Earls on April 17, 2020
  • December 2017 LSAT
  • SEC1
  • Q16
2
Replies
Help Eliminate answer choice A
I chose answer A because I interpreted it to mean an alternative reason why Chu will win. Please ...
Ugo-Anyanwu on January 25, 2020
  • December 2017 LSAT
  • SEC1
  • Q1
1
Reply